Difference between revisions of "1993 AIME Problems/Problem 4"

(Solution 3)
Line 41: Line 41:
  
 
Thus, the answer is <math>405 + 465 = \boxed{870}</math>
 
Thus, the answer is <math>405 + 465 = \boxed{870}</math>
 +
 +
===Solution 4===
 +
 +
Add the two conditions together to get <math>a+d+ad+93=b+c+bc</math>. Rearranging and factorising with SFFT, <math>(a+1)(d+1)+93=(b+1)(c+1)</math>. This implies that for every quadruple <math>(a,b,c,d)</math>, we camp replace <math>a\longrightarrow a+1</math>, <math>b\longrightarrow b+1</math>, etc. and this will still produce a valid quadruple. This means, that we can fix <math>a=1</math>, and then just repeatedly add <math>1</math> to get the other quadruples.
 +
 +
Now, our conditions are <math>b+c=d+1</math> and <math>bc=d+93</math>. Replacing <math>d</math> in the first equation, we get <math>bc-b-c=92</math>. Factorising again with SFFT gives <math>(b-1)(c-1)=93</math>. Since <math>b<c</math>, we have two possible cases to consider.
 +
 +
Case 1: <math>b=2</math>, <math>c=94</math>. This produces the quadruple <math>(1,2,94,95)</math>, which indeed works.
 +
 +
Case 2: <math>b=4</math>, <math>c=32</math>. This produces the quadruple <math>(1,4,32,35)</math>, which indeed works.
 +
 +
Now, for case 1, we can add <math>1</math> to each term exactly <math>404</math> times (until we get the quadruple <math>(405,406,498,499)</math>), until we violate <math>d<500</math>. This gives <math>405</math> quadruples for case 1.
 +
 +
For case 2, we can add <math>1</math> to each term exactly <math>464</math> times (until we get the quadruple <math>(465,468,496,499)</math>). this gives <math>465</math> quadruples for case 2.
 +
 +
In conclusion, having exhausted all cases, we can finish. There are hence <math>405+465=\boxed{870}</math> possible quadruples.
  
 
== See also ==
 
== See also ==

Revision as of 04:02, 4 July 2018

Problem

How many ordered four-tuples of integers $(a,b,c,d)\,$ with $0 < a < b < c < d < 500\,$ satisfy $a + d = b + c\,$ and $bc - ad = 93\,$?

Solution

Solution 1

Let $k = a + d = b + c$ so $d = k-a, b=k-c$. It follows that $(k-c)c - a(k-a) = (a-c)(a+c-k) = (c-a)(d-c) = 93$. Hence $(c - a,d - c) = (1,93),(3,31),(31,3),(93,1)$.

Solve them in tems of $c$ to get $(a,b,c,d) = (c - 93,c - 92,c,c + 1),$ $(c - 31,c - 28,c,c + 3),$ $(c - 1,c + 92,c,c + 93),$ $(c - 3,c + 28,c,c + 31)$. The last two solutions don't follow $a < b < c < d$, so we only need to consider the first two solutions.

The first solution gives us $c - 93\geq 1$ and $c + 1\leq 499$ $\implies 94\leq c\leq 498$, and the second one gives us $32\leq c\leq 496$.

So the total number of such four-tuples is $405 + 465 = \boxed{870}$.

Solution 2

Let $b = a + m$ and $c = a + m + n$. From $a + d = b + c$, $d = b + c - a = a + 2m + n$.

Substituting $b = a + m$, $c = a + m + n$, and $d = b + c - a = a + 2m + n$ into $bc - ad = 93$, \[bc - ad = (1 + m)(1 + m + n) - a(a + 2m + n) = m(m + n). = 93 = 3(31)\] Hence, $(m,n) = (1,92)$ or $(3,28)$.

For $(m,n) = (1,92)$, we know that $0 < a < a + 1 < a + 93 < a + 94 < 500$, so there are $405$ four-tuples. For $(m,n) = (3,28)$, $0 < a < a + 3 < a + 31 < a + 34 < 500$, and there are $465$ four-tuples. In total, we have $405 + 465 = \boxed{870}$ four-tuples.

Solution 3

Square both sides of the first equation in order to get $bc$ and $ad$ terms, which we can plug $93$ in for.

$(a+d)^2 = (b+c)^2 \implies a^2 + 2ad + d^2 = b^2 + 2bc + c^2 \implies 2bc-2ad = a^2-b^2 + d^2-c^2 \implies 2(bc-ad) = (a-b)(a+b)+(d-c)(d+c)$

We can plug $93$ in for $bc - ad$ to get $186$ on the left side, and also observe that $a-b = c-d$ after rearranging the first equation. Plug in $c-d$ for $a-b$.

$186 = (c-d)(a+b) + (d-c)(d+c) \implies 186 = -(d-c)(a+b) + (d-c)(d+c) \implies 186 = (d-c)(d+c-a-b)$

Now observe the possible factors of $186$, which are $1 \cdot 186, 2\cdot 93, 3 \cdot 62, 6\cdot 31$. $(d-c)$ and $(d+c-a-b)$ must be factors of $186$, and $(d+c-a-b)$ must be greater than $(d-c)$.

$1 \cdot 186$ work, and yields $405$ possible solutions. $2 \cdot 93$ does not work, because if $c-d = 2$, then $a+b$ must differ by 2 as well, but an odd number $93$ can only result from two numbers of different parity. $c-d$ will be even, and $a+b$ will be even, so $c+d - (a+b)$ must be even. $3 \cdot 62$ works, and yields $465$ possible solutions, while $6 \cdot 31$ fails for the same reasoning above.

Thus, the answer is $405 + 465 = \boxed{870}$

Solution 4

Add the two conditions together to get $a+d+ad+93=b+c+bc$. Rearranging and factorising with SFFT, $(a+1)(d+1)+93=(b+1)(c+1)$. This implies that for every quadruple $(a,b,c,d)$, we camp replace $a\longrightarrow a+1$, $b\longrightarrow b+1$, etc. and this will still produce a valid quadruple. This means, that we can fix $a=1$, and then just repeatedly add $1$ to get the other quadruples.

Now, our conditions are $b+c=d+1$ and $bc=d+93$. Replacing $d$ in the first equation, we get $bc-b-c=92$. Factorising again with SFFT gives $(b-1)(c-1)=93$. Since $b<c$, we have two possible cases to consider.

Case 1: $b=2$, $c=94$. This produces the quadruple $(1,2,94,95)$, which indeed works.

Case 2: $b=4$, $c=32$. This produces the quadruple $(1,4,32,35)$, which indeed works.

Now, for case 1, we can add $1$ to each term exactly $404$ times (until we get the quadruple $(405,406,498,499)$), until we violate $d<500$. This gives $405$ quadruples for case 1.

For case 2, we can add $1$ to each term exactly $464$ times (until we get the quadruple $(465,468,496,499)$). this gives $465$ quadruples for case 2.

In conclusion, having exhausted all cases, we can finish. There are hence $405+465=\boxed{870}$ possible quadruples.

See also

1993 AIME (ProblemsAnswer KeyResources)
Preceded by
Problem 3
Followed by
Problem 5
1 2 3 4 5 6 7 8 9 10 11 12 13 14 15
All AIME Problems and Solutions

The problems on this page are copyrighted by the Mathematical Association of America's American Mathematics Competitions. AMC logo.png